You are on page 1of 45

Table

 of  Contents  
1.INTRODUCTION  AND  SOURCES  OF  THE  CRIMINAL  LAW  .......................................................  3  
A)  THE  COURT  SYSTEM  ............................................................................................................................................  3  
B)  SOURCES  OF  CRIMINAL  LAW  ..............................................................................................................................  3  
i)  Switzman  v.  Elbling  SCC  [1957]  ..................................................................................................................  4  
ii)  R.  v.  Morgentaler  [1993]  S.C.C  ....................................................................................................................  4  
iii)  Reference  re  Firearms  Act  [2000]  SCC  ...................................................................................................  4  
2.CODIFICATION  AND  INTERPRETATION  ......................................................................................  5  
A)  ORIGINS  OF  THE  CODE  .........................................................................................................................................  5  
B)  THE  CODE  ..............................................................................................................................................................  5  
i)  Amato  v.  The  Queen  [1982]  SCC  ..................................................................................................................  5  
ii)  Frey  v.  Fedoruk  Supreme  Court  of  Canada  [1950]  S.C.C  .................................................................  5  
iii)  Jobidon  v.  The  Queen  [1991]  SCC  .............................................................................................................  5  
C)  INTERPRETATION  –  VAGUENESS,  OVERBREADTH,  AND  CERTAINTY  ..........................................................  6  
i)  R.  v.  Nova  Scotia  Pharmaceutical  Society  [1992]  SCC  .......................................................................  6  
ii)  R.  v.  Heywood  [1994]  SCC  .............................................................................................................................  7  
ii)  Canadian  Foundation  for  Children,  Youth,  and  the  Law  v.  Canada  [2004]  SCC  ...................  7  
D)  INTERPRETATION  –  DOCTRINE  OF  STRICT  CONSTRUCTION  ........................................................................  8  
i)  R.  v.  Pare  [1987]  SCC  ........................................................................................................................................  8  
ii)  Bell  ExpressVu  Limited  Partnership  v.  Rex  [2002]  SCC  ...................................................................  8  
3.THE  CHARTER  AND  CRIMINAL  LAW  .............................................................................................  9  
A)  INTRODUCTION  TO  THE  CHARTER  .....................................................................................................................  9  
B)  THE  OAKES  TEST  ..................................................................................................................................................  9  
i)  R.  v.  Oakes  Supreme  Court  of  Canada  [1986]  SCC  ...............................................................................  9  
4.EXCLUSION  OF  EVIDENCE  UNDER  THE  CHARTER  ..................................................................  10  
A)  PRE-­‐CHARTER  EXCLUSIONS  ............................................................................................................................  10  
B)  PRE-­‐GRANT  EXCLUSIONS  .................................................................................................................................  10  
i)  R.  v.  Collins  Supreme  Court  of  Canada  [1987]  SCC  ..........................................................................  10  
ii)  R.  v.  Stillman  Supreme  Court  of  Canada  [1997]  SCC  .....................................................................  11  
iii)  R.  v.  Buhay  2003  SCC  ..................................................................................................................................  11  
C)  THE  CHARTER  AND  SECTION  24(2):  EXCLUDING  EVIDENCE  ...................................................................  11  
GRANT  TEST:  ............................................................................................................................................................  11  
i)  R.  v.  Grant,  Supreme  Court  of  Canada  [2009]  SCC  ...........................................................................  11  
ii)R.  v.  Harrison,  Ontario  Court  of  Appeal  [2008]  ONCA  85  .............................................................  12  
5.  CRIMINAL  PROCEDURE  1  ...............................................................................................................  13  
A)  ELEMENTS  AND  COMPONENTS  OF  THE  OFFENCE  ........................................................................................  13  
B)  CHARGE  APPROVAL  STANDARDS  ....................................................................................................................  14  
C)  INFORMATIONS  AND  INDICTMENTS  ................................................................................................................  14  
6.  INTRODUCTION  TO  DUE  PROCESS  .............................................................................................  15  
A)  POLICE  POWERS  ................................................................................................................................................  15  
B)  MODELS  OF  THE  CRIMINAL  PROCESS  ............................................................................................................  15  
C)  THE  CHARTER  .....................................................................................................................................................  15  
7.  THE  LAW  OF  SEARCH  AND  SEIZURE  ..........................................................................................  16  
A)  ORIGINS  ...............................................................................................................................................................  16  

1
B)  SOURCES  OF  SEARCH  POWER  ..........................................................................................................................  16  
C)  CHALLENGING  SEARCHES  .................................................................................................................................  16  
Challenging  Warranted  Searches:  .........................................................................................................  17  
i)  Hunter  v.  Southam  Inc.  Supreme  Court  of  Canada  [1984]  SCC  ..................................................  17  
D)  REASONABLE  EXPECTATION  OF  PRIVACY  .....................................................................................................  18  
i)  R.  v.  Edwards  Supreme  Court  of  Canada  [1996]  SCC  ......................................................................  19  
ii)R.  v.  Wong  Supreme  Court  of  Canada  [1990]  SCC  ............................................................................  19  
iii)  R.  v.  Plant  Supreme  Court  of  Canada,  [1993]  SCC  .........................................................................  20  
iv)  R.  v.  Tessling  [2004]  SCC  ...........................................................................................................................  20  
v)  R  v  Patrick,  Supreme  Court  of  Canada  [2009]  SCC  .........................................................................  21  
8.  ARREST  ................................................................................................................................................  21  
A)  POLICE  INTERACTIONS  WITH  CITIZENS  ........................................................................................................  21  
B)  ARREST  WITHOUT  WARRANT  .........................................................................................................................  22  
i)  R  v.  Storrey  [1990]  SCC  ................................................................................................................................  22  
C)  ARREST  WITH  WARRANT  .................................................................................................................................  23  
9.  ANCILLARY  POWERS:  INCIDENT  TO  ARREST  AND  POWER  OF  DETENTION  .................  24  
A)  ANCILLARY  POWERS  INCIDENT  TO  ARREST  .................................................................................................  24  
i)  Cloutier  v.  Langlois  [1990]  SCC  ................................................................................................................  24  
ii)  R  v.  Golden  [2001]  SCC  ................................................................................................................................  25  
iii)  R.  v.  Caslake,[1998]  SCC  ............................................................................................................................  25  
B)  POWERS  OF  DETENTION  ..................................................................................................................................  26  
i)  R.  v.  Simpson,  1993  (ON  CA)  ......................................................................................................................  26  
ii)  R.  v.  Mann,  [2004]  SCC  ................................................................................................................................  27  
iii)  R.  v.  Clayton,  [2007]  SCC  ...........................................................................................................................  28  
C)  DETAINMENT  ......................................................................................................................................................  28  
i)  R  v  Grant  [2009]  SCC  .....................................................................................................................................  29  
ii)  R  v  Suberu,  [2009]  SCC  ................................................................................................................................  29  
10.  CONFESSION  ....................................................................................................................................  30  
A)  ADMISSIBILITY  OF  CONFESSIONS  ....................................................................................................................  31  
i)  Boudreau  v.  The  King,  SCC  [1949]  S.C.C  ................................................................................................  31  
ii)  R  v  Wray  SCC  [1971]  ....................................................................................................................................  31  
iv)  Rothman  v  The  Queen,  SCC  [1981]  SCC  ..............................................................................................  32  
B)  EVALUATING  VOLUNTARINESS  ........................................................................................................................  32  
11.  CONFESSIONS:  COMMON  LAW  RULES  .....................................................................................  33  
i)  R.  v.  Oickle  SCC  [2000]  ..................................................................................................................................  33  
ii)  R.  v.  Spencer  SCC  [2007]  .............................................................................................................................  33  
12.  CONFESSIONS:  THE  RIGHT  TO  SILENCE  AND  RIGHT  TO  COUNSEL  ................................  34  
i)  Clarkson  v  The  Queen,  SCC  [1986]  ..........................................................................................................  34  
ii)  R.  v.  Manninen  SCC  [1987]  ........................................................................................................................  34  
iii)  R.  v.  Brydges  SCC  (1990)  ..........................................................................................................................  35  
iv)  R.  v.  Prosper  SCC  [1994]  ...........................................................................................................................  35  
v)  R.  v.  Smith  SCC  [1989]  .................................................................................................................................  35  
vi)  R.  v.  Hebert  SCC  [1990]  .............................................................................................................................  36  
vii)  R.  v.  Singh  SCC  [2007]  ..............................................................................................................................  36  
viii)  R.  v.  Sinclair  SCC  [2010]  .........................................................................................................................  37  
13.  BAIL  AND  OTHER  PRE-­‐TRIAL  MATTERS  ................................................................................  38  
INTRO  TO  BAIL,  MULTIPLE  INFOS,  WAIVERS  AND  PARTIES  ...........................................................................  38  

2
R  v  Dunlop  and  Sylvester  SCC  1979  (Who  is  a  party  to  a  crime)  ...................................................  38  
BAIL  (JUDICIAL  INTERIM  RELEASE)  ....................................................................................................................  38  
ONUS,  BURDEN  OF  PROOF,  STANDARD  OF  PROOF  AND  BREACHES  OF  BAIL  ...............................................  39  
ARRAIGNMENT  AND  PLEA  .....................................................................................................................................  39  
ELECTIONS,  PRELIMINARY  HEARINGS,  AND  DELAY  ..........................................................................................  40  
i)  R  v  Morin  SCC  1992  ........................................................................................................................................  40  
14.  SENTENCING  ....................................................................................................................................  41  
MECHANICS  OF  SENTENCING:  RANGE  AND  OPTIONS  .......................................................................................  41  
PRINCIPLES  OF  SENTENCING  .................................................................................................................................  41  
i)  Gladue  v  The  Queen  SCC  1999  ...................................................................................................................  41  
ii)  R  v  Ipeelee  and  R  v  Ladue  SCC  2012  ......................................................................................................  42  
iii)  The  Queen  v  L.M  SCC  2008  (Test  for  when  a  sentence  can  be  varied)  ..................................  42  
iv)  R  v  Smith  SCC  1987  (When  is  there  a  violation  of  s.12)  ..............................................................  42  
JURIES  ........................................................................................................................................................................  43  
CHALLENGES  ............................................................................................................................................................  43  
THE  ROLE  OF  COUNSEL  ..........................................................................................................................................  44  
i)  R  v  Boucher  SCC  [1955]  (Crown  told  jury  he  was  guilty,  Crown  is  not  out  to  obtain  a  
guilty  verdict)  .......................................................................................................................................................  44  
ii)  R  v  Stinchcomb  SCC  [1991]  (the  Duty  to  disclose  all  relevant  information)  .......................  44  
ROLE  OF  DEFENCE  ..................................................................................................................................................  45  

1.Introduction  and  Sources  of  the  Criminal  Law  


A)  The  Court  System    
The Rule of Law: “No one is above the law and everyone is equal before the law”
• Criminal law starts with a decision of a legislature to define something as
prohibited.
• Provincial courts: deal w/both criminal and civil matters (Superior Provincial
Courts deal with civil matters of any amount, large family matters, and serious
criminal matters i.e. murder and Provincial Courts deal with criminal matters and
small claims up to 25K small family legal matters)
• Federal courts deal w/a few ‘public law’ matters

B)  Sources  of  Criminal  Law  


1. The Constitution – The Constitution Act of 1867/1982, the Charter.
2. Federal Legislation – The Criminal Code, Youth Criminal Justice Act, etc. etc.
3. Case Law
4. Treaties and Conventions, administrative tribunal decisions.

3
i) Switzman v. Elbling SCC [1957]
Ratio: Parliament has exclusive legislative authority over matters of criminal law.
Any law, enacted by provincial legislatures, found to deal w/matters of criminal law
will be found unconstitutional.

Facts: The gov’t of QB attempted to enact a statute making it illegal for any person who
owns a house to use that house to propagate communism or bolshevism. The statute stated
that if you broke the law, the government could take possession of your house and, if you
attempted to back into the house, they could put you in jail.

Issue: Is the Padlock Act ultra vires to the power of the provincial government?

Conclusion: The substance of the Act is criminal, therefore it is inevitable that the Act is
unconstitutional.
ii) R. v. Morgentaler [1993] S.C.C
Ratio: Parliament has exclusive legislative authority over matters of criminal law.
Any law, enacted by provincial legislatures, found to deal w/matters of criminal law
will be found to be unconstitutional.

Facts: The Nova Scotia Medical Services Act, made it an offense to perform an abortion
outside a hospital in Nova Scotia.

Issue: Do the regulations in The Nova Scotia Medical Services Act, overstep the
provincial jurisdiction because they deal w/matters of criminal law?

Conclusion: The substance of the Act is criminal law, therefore the Act is
unconstitutional.
iii) Reference re Firearms Act [2000] SCC
Ratio: Parliament has exclusive legislative authority over matters of criminal law.
Any law, enacted by provincial legislatures, found to deal w/matters of criminal law
will be found to be unconstitutional.

Facts: Parliament amended the Criminal Code to require the holder of firearms to obtain
licenses and register their guns. AB challenged this.

Issue: Does the Parliament have jurisdiction to enact a law over the regulation of firearms
in the Provinces?

Conclusion: Supreme Court held that parliament could require gun owners to obtain
licenses and to register guns under its criminal law power, and the legislation did not
involve the regulation of property and civil rights under s. 92(13) of the province as the
province had contended.

4
2.Codification  and  Interpretation  
A)  Origins  of  the  Code  
i) 1953, all common law offenses abolished in Canada in s.9, s.8(3) preserved all
existing common law defences not inconsistent w/the code.
ii) Debate about the nature of the Code; Bentham v. Stephen – Bentham more
concerned w/ deterrence based on utilitarian views, Stephen more concerned
w/ retribution based on morality. --> Stephen won the day.

B)  The  Code    
i) Amato v. The Queen [1982] SCC
Ratio: s.8(3) did not preclude the courts from recognizing new criminal defenses as
long as they are consistent with the provisions set out by the Code.

Facts: The appellant was convicted of trafficking cocaine after an undercover officer
repeatedly and persistently requested drugs from appellant despite his lack of interest.

Issue: Did s.8(3) of the Criminal Code preserve old defenses only or are the courts
allowed to make new ones such as the defense of entrapment?

Conclusion: Held that common law would be allowed to develop defenses not
inconsistent with the provisions of the Code if the construction adopted was prospective.
However, it was recognized that ultimate power over this area was still held by the
legislature and that evidence of legislative intent was needed to support this new defense.
(Note: in this case entrapment was not held to be a valid defense for this particular
defendant.)
ii) Frey v. Fedoruk Supreme Court of Canada [1950] S.C.C
Ratio: s.9 of the Code abolished all common law offences and precluded the
development of any new offences by the courts.

Facts: Frey was caught peering into a women’s window and detained by Fedoruk. The act
of peeping was not a criminal offense at the time..

Issue: Should the courts recognize new criminal offences under common law such as
“Peeping Tom”?

Conclusion: Held not to increase the number of common law offenses and thus refused to
declare it to be an offence to be a peeping tom. The court felt that allowing common law
offenses to arise through the judiciary process would result in the introduction of great
uncertainty into the administration of the Criminal Law. This was not a new offence,
therefore it was a unlawful detainment.
iii) Jobidon v. The Queen [1991] SCC
Ratio: Even though the Judiciary cannot create new Criminal offences, they still
have the discretion to interpret existing offences laid out in the Criminal Code.

5
Facts: Jobidon was charged w/manslaughter after a fight he had resulted in other parties
death. The fight was consensual but for public policy reasons the court held there could be
no consent to such an assault resulting in death.

Issues: Can the courts read a lack of implied consent into the definition of assault? If the
court does so are they creating a new offence?

Conclusions: Court can read limitations into the definition of consent. This doesn’t
amount to creating a new offence; it is just explaining the outline and boundaries of the
existing criminal offenses. In this case, there was a limit to the victims consent set out by
customary norms. Also, common law can still inform interpretations of the criminal code.
In general, despite all offences in Canada being defined in s.9 of the Code, common law
still can illuminate these definitions.

C)  Interpretation  –  Vagueness,  Overbreadth,  and  Certainty  


• Fundamental principle: “no crime without law, no punishment without law”
• Captured in s. 11(g) and (i) of Charter
• (g) not to be found guilty on account of any act or omission unless, at the time of the
act or omission, it constituted an offence under Canadian or international law or was
criminal according to the general principles of law recognized by the community of
nations
• (i) if found guilty of the offence and if the punishment for the offence has been
varied between the time of commission and the time of sentencing, to the benefit of
the lesser punishment.
• supported in contemporary criminal law by doctrines of vagueness and overbreadth

Void for Vagueness Doctrine


• Principle of fundamental justice that vague laws are unconstitutional as per s.7 of the
Charter “Everyone has the right to life, liberty and security of the person and the
right not to be deprived thereof except in accordance with the principles of
fundamental justice.”
• A law is unconstitutional due to “vagueness” when… meaning of a law can’t be
clearly ascertained
i) R. v. Nova Scotia Pharmaceutical Society [1992] SCC
Ratio: S.7 asserts the principle of fundamental justice, laws may not be too vague. If
they are found too vague, they will be unconstitutional. Test for Vagueness.

Facts: A number of pharmacies were charged with conspiracy "to prevent or lessen
competition" under section 32(1)(c) of the Combines Investigation Act for the sale of
prescription drugs and dispensing services prior to June 1986. They challenged the
provision on the basis that it violated section 7 of the Charter on account of its vagueness.

Issue: Was s. 32(1)(c) of the Combines Investigation Act too vague and, if so, in breach of
s.7 of the Charter?

6
Conclusion: Courts recognized that s.7 gives rise to the Void for Vagueness Doctrine
under which laws that are too vague will be found to be unconstitutional. The courts laid
out a three-part test to determine if a law is vague (quite an easy test to past)

Test for Vagueness:


• if the law does not provide adequate basis for legal debate and analysis
• if the law does not sufficiently delineate an area of risk and
• and if the law is not intelligible

Note: In most cases, challenges under s.7 have failed w/courts stressing that they can use
limiting interpretations on expansively worded statements.

Void for Overbreadth Doctrine


1. Principle of fundamental justice that overbreadth laws are unconstitutional as per
s.7.
2. A law is unconstitutional due to “overbreadth” when…laws are too sweeping in
scope
ii) R. v. Heywood [1994] SCC
Ratio: S.7 principle of fundamental justice that laws may not be too overbroad. If
they are too overbroad, they will be unconstitutional. Test for overbroad.

Facts: Heywood was convicted under section 246.1(1) for sexual assault of children. The
conviction made him subject to section 179(1)(b), which prevented convicted individuals
from loitering “in or near a school ground, playground, public park or bathing area.”

Issue: Was s.179(1)(b) of the Criminal Code overbroad and, if so, in breach of s.7?

Conclusion: The courts recognized that s.7 of the Charter gives rise to the Void for
Overbreadth Doctrine. Overbreadth analysis looks at the means chosen by the state in
relation to its purpose. If the state, in pursuing a legitimate objective, uses means, which
are broader than is necessary to accomplish the objective, the principles of fundamental
justice will be violated. The courts outlined a two part test;

Two-stage test for whether law is overbroad:


1) What is the objective of the law?
2) Are the means chosen necessary to the objective?

In this case, s. 179(1)(b) was deemed overbroad and was found to be unconstitutional
under s.7 of the Charter. The court specifically referenced that: geographical was too
broad, temporally too broad, and too broad in the number of people it covered.
ii) Canadian Foundation for Children, Youth, and the Law v. Canada [2004] SCC
Ratio: As per s.7, it is a principle of fundamental justice that laws may not be too
overbroad or too vague. If found to be either, they will be unconstitutional.

7
Facts: The Canadian Foundation applied for a declaration to strike-down s.43 of the Code
which authorized the use of force “by way of correction toward a pupil or child...if the
force does not exceed what is reasonable under the circumstances” b/c it was too vague
and overbroad.

Issue: Was s.43 of the Code too vague and too overbroad and, if so, in violation of s.7 of
the Charter?

Conclusion: Held that the section was neither too vague nor too broad and was therefore
constitutional. Court stated that the section set a standard often seen in criminal law of
reasonableness and drew on expert opinion on what is acceptable. A ‘solid core of
meaning’ was inferred in s. 43. As well, even though it was argued that s. 43 was
overbroad because it applies to children under 2 and over 12 who should not be
reasonably included, the court held that s. 43 should be read so as not to include them.
Note: Dissent Arbour J. felt this restrictive interpretation can only be arrived at if dictated
by constitutional imperatives. She felt the courts stretched too far in their interpretation
and read too much in to save the section.

D)  Interpretation  –  Doctrine  of  Strict  Construction    


Doctrine of Strict Construction: Traditionally, criminal/penal statutes were strictly
construed in favor of the accused; however, recent cases hold strict construction applies
only if ambiguity remains after provision is interpreted in light of larger purpose/objective
of statute.
i) R. v. Pare [1987] SCC
Ratio: In resolving statute ambiguities, Doctrine of Strict Construction will only
apply if the ambiguity remains after the statute is interpreted in light of the larger
purpose it was meant to serve.

Facts: s. 214(5)(b) makes murder 1st degree if death was caused “while committing” an
indecent assault. Pare, a 17-year-old, was accused of murdering a 7-year-old boy. The
murder happened 2 minutes after the 17-year-old accused indecently assaulted the boy.

Issue: In section 214(5)(b), does “while committing” require that murder happen at the
same time the accused commits indecent act?

Conclusion: The courts state that taking a strict construction approach in this case raises
two problems: when did the assault begin and end, and leads to arbitrary distinctions. A
common sense, alternate reading is more consistent w/policy underlying section namely a
more severe punishment for more serious conduct. Therefore, the single transaction
analysis was deemed the proper construction of s.214(5) and Pare convicted of 1st degree.
ii) Bell ExpressVu Limited Partnership v. Rex [2002] SCC
Ratio: Courts will have to resort to strict construction when a provision is
“reasonably capable of more than one meaning” and each meaning is consistent
w/the intent of the statute.

8
3.The  Charter  and  Criminal  Law  
A)  Introduction  to  the  Charter    
• from 1867 onwards, Canada has had a constitution “similar in principle to that of the
United Kingdom”
• rights protected in UK’s “unwritten Constitution”: i.e., Magna Carta, Bill of Rights
1689, common law on habeas corpus…
• 1982 Charter of Rights is an “entrenched” bill of rights.
• Generally speaking the Charter is meant to limit the governments actions
• Powers of enforcement: s. 24: right for individuals who have had their rights violated
to obtain a remedy the court considers appropriate and right of courts to exclude
evidence if obtained in a way that violated someone’s Charter rights.
• s.7: “Everyone has the right to life, liberty and security of the person and the right
not to be deprived thereof except in accordance with the principles of fundamental
justice.”
• Fundamental justice come from s.8 – s.14 but has other common law definitions as
well (i.e. vagueness and overbreadth)
Limits to the Charter
• s.1 “guarantees the rights and freedoms set out in it subject only to such reasonable
limits prescribed by law as can be demonstrably justified in a free and democratic
society.”
• S.32 Charter rights only apply between individuals and the government.
• S.33 Notwithstanding clause, governments can basically pause any of the rights in
the charter and unilaterally suspend them—politically unusable.

B)  The  Oakes  Test  


1) Objective of leg/provision must be of sufficient importance – “pressing and
substantial”
2) (Proportionality test) The means chosen must be reasonably and demonstrably
justified which means
• means must be rational connection to objective
• means must impair “as little as possible”
• must be proportionality between effects and objective (benefit outweigh the
deleterious effects)
i) R. v. Oakes Supreme Court of Canada [1986] SCC
Ratio: Laws found to infringe a Charter right can upheld as constitutional if the
limiting of the right can be saved under s.1 of the Charter. i.e. the limit needs to be
demonstrably justified in a free and democratic society.

Facts: The constitutionality of S. 8 of the Narcotic Control Act was challenged. The law
stated if the Court finds the accused guilty of possession of a narcotic, the accused is
presumed to be in possession for the purposes of trafficking and that, absent the accused’s
dispelling this, he must be convicted of trafficking. This is a reverse onus clause in which
it becomes the accused responsibility to absolve himself of guilt.

9
Issue: Does s. 8 of the Narcotic Control Act violate s.11(d) of the Charter and, if it does,
is s.8 a reasonable limit prescribed by laws as can be demonstrably justified in a free and
democratic society for the purpose of s.1 of the Charter?

Conclusion: The court found s.8 did violate s.11(d) of the Charter because;
1. a provision which requires an accused to disprove on a balance of probabilities the
existence of a presumed fact, violates the presumption of innocence in s.11(d) of
the Charter and,
2. The limitation of this right could not be shown by the Crown to (a) have an
objective that is sufficiently important to warrant overriding a constitutionally
protected right or freedom and (b) impair the right in question as little as possible.

Courts found s. 8 doesn’t survive the rational connection test. Possession of a small
quantity of narcotics doesn’t support the inference of trafficking. The legislation was
struck down.

4.Exclusion  of  Evidence  Under  the  Charter  


A)  Pre-­‐Charter  Exclusions  
• Prior to Charter, courts refused to reject evidence on the basis of how was obtained.
Had no power to exclude if you were treated horribly.
• Exception to this: at common law, confessions excluded automatically unless the
crown could prove voluntariness Beyond A Reasonable Doubt (BARD).
• Common law approach to illegally obtained evidence focussed on the reliability, not
fairness of process leading to discovery (e.g., R v. Wray)

B)  Pre-­‐Grant  Exclusions    
s. 24(2) Where…a court concludes that evidence was obtained in a manner that infringed
or denied any rights or freedoms guaranteed by this Charter, the evidence shall be
excluded if it is established that, having regard to all the circumstances, the admission of
it in the proceedings would bring the administration of justice into disrepute.

i) R. v. Collins Supreme Court of Canada [1987] SCC


Rule: In Collins, the Court held that for a search to be reasonable, it had to be:
1. Authorized by law (statutory or common law);
2. The authorizing law has to be reasonable (Hunter); and
3. The authorized search has to be carried out reasonably.

Facts: Suspects were thought to be trafficking. Police officer was very rough in the arrest.

Collins established a three part test for exclusion of evidence:


• Would admitting the evidence affect trial fairness?
• How serious was the Charter breach?
• Would the exclusion of the evidence bring the administration of justice into
disrepute?

10
ii) R. v. Stillman Supreme Court of Canada [1997] SCC
Rule: Refined 1st branch of Collins (trial fairness), evidence would affect trial
fairness if it is conscriptive and non-discoverable.

Facts: Suspect, 17 yrs old, accused of murdering a 14 year old girl. Police refused his
right to talk to a lawyer. Forcibly take body samples from him, dental etc.

Analysis: Stillman refined first branch of Collins Test (i.e. trial fairness)
• conscriptive - accused is compelled to provide a statement, a bodily sample, etc.,
which incriminates. Evidence that results from a compelled statement etc is
referred to as derivative --> can only be saved if it was otherwise discoverable.
• “non-discoverable” - if it could not been obtained “but for” the Charter
violation. Crown can prove “discoverability” by establishing, on a balance of
probabilities: i)There was an alternate means of finding the evidence ii)You would
find it inevitably. Court found a pattern of abuse.
iii) R. v. Buhay 2003 SCC
Rule: Elaborates 2nd and 3rd branches of Collins: seriousness of breach (trivial v
technical) and effects of exclusion on Crowns case/seriousness of offence.

Facts: Police officers could smell marijuana in a locker. One police officer never even
thought to get a warrant and the second thought you did not have reasonable right to
privacy in a public locker

Buhay elaborated upon the 2nd and 3rd Branches of the Collins framework:
2. Seriousness of the Charter Breach
- refers to the seriousness of the police’s violation
- i.e., is it trivial or technical versus deliberate
3. Effect of Exclusion on the Reputation of Admin of Justice
- involves consideration of seriousness of offence and importance of evidence to
Crown’s case.
Note: These three cases are no longer considered good law, Grant is the new authority on
this matter.

C)  The  Charter  and  Section  24(2):  Excluding  Evidence  


Grant  Test:  
• Consider the seriousness of the Charter infringing state conduct;
• Assess the impact of the breach on the Charter protected interests of the
accused; and
• Consider society’s interest in the adjudication of the case on its merits.
i) R. v. Grant, Supreme Court of Canada [2009] SCC
Ratio: When evidence is obtained in a way in which a person’s Charter rights are
violated, courts will consider the seriousness of the infringing state conduct, the
impact of the breach on the protected interests of the accused, and societies interest

11
in the adjudication of the case on its merits to determine if the evidence will be
excluded under s.24 of the Charter.

Facts: 3 police officers were patrolling a school area known for a high crime rate. They
observed Grant in the area, acting suspiciously. They approached him, blocked his path
and began questioning him. Before arresting him, one of the 3 officers asked him if he
had anything on him that he shouldn’t to which he stated he had some marijuana and a
loaded gun. They arrested him and advised him of his right to council.

Issue: Whether the evidence was obtained in breach of Grant’s Charter rights and if so,
whether it should be excluded under s.24(2)?

Conclusion: The courts felt that the central question animating the section 24(2) inquiry
is: “whether a ROP, informed of all the relevant circumstances and values underlying the
Charter could conclude that the admission of evidence would bring the administration of
justice into disrepute?”
This involves new 3-stage test:
1. Consider the seriousness of the Charter infringing state conduct;
• Was the conduct inadvertent or minor?
• Were there extenuating circumstances?
• Or was it wilful, reckless or deliberate?
• Was it part of a pattern of abuse? (ex Stillman)
2. Assess the impact of the breach on the Charter protected interests of the accused;
• Which Charter interests are engaged by the infringement? (e.g. s. 7, 8, 9,
10a, 10b)
• To what extent were those interests violated? Ie) body cavity v bus locker
3. Consider society’s interest in the adjudication of the case on its merits. (Would
the truth-seeking function of the criminal trial process be better served by
admission of the evidence or by its exclusion?) Reliability, importance to
Crown’s case, is the offence serious.

Courts found there was a violation of Grants Charter rights and, but that the gun should
not be excluded as evidence against Mr. Grant b/c the state conduct wasn’t very serious.
The rights infringed were significant but not too serious and the gun was highly reliable
for a serious charge (very close call though).
ii)R. v. Harrison, Ontario Court of Appeal [2008] ONCA 85
Ratio: When evidence is obtained in a way that violates someone’s Charter rights,
courts will consider the seriousness of the infringing state conduct, the impact of the
breach on the protected interests of the accused, and societies interest in the
adjudication of the case on its merits to determine if the evidence will be excluded
under s.24 of the Charter.

Facts: The police officer was suspicious of Harrison’s vehicle when he saw no front
license plate. Even after realizing the car was from AB and didn’t need a front plate, the
officer still decided to pull Harrison over. Harrison’s car was searched w/o cause, as was a

12
sealed box in the back seat. Inside the box they discovered $2.4 to 4.6 million dollars
worth of cocaine. The courts found a breach of s.8 (search) and s.9 (detention) – 9 no
reason to pull over the vehicle, 8 no reason to go into the car or open the box.

Issue: Should the evidence be excluded because they were obtained in a way that violated
Harrison’s Charter rights?

Conclusion: At trial it was admitted on appeal the SCC held that the evidence should
have been excluded. The Judge used the Grant test to determine if the evidence should
have been excluded.
• Seriousness of the Charter-infringing state conduct:
• the police conduct in stopping and searching his vehicle w/o any semblance of
reasonable founds was reprehensible. (Favors’ exclusion)
• Impact on the Charter protected interest of the accused:
• The infringement had a significant, although not egregious, impact on the
Charter protected interests of the Accused. (Favors’ exclusion)
• Society’s interest in an adjudication on the merits:
• Drugs seized are highly reliable evidence for a very serious charge. (Favours
Inclusion)

It is a test done by qualitative balancing, each aspect must be weighed to determine


whether, having regard to all the circumstances, admission of the evidence would bring
the administration of justice into disrepute.
In this case, the police misconduct was serious; indeed, the trial judge found that it
represented a “brazen and flagrant” disregard of the Charter. To appear to condone willful
and flagrant Charter breaches that are a significant incursion on an appellant’s rights
doesn’t enhance the long-term repute of the administration of justice; on the contrary, it
undermines it. In this case, the seriousness of the offence and the reliability of the
evidence, while important, do not outweigh the factors pointing to exclusion.

5.  Criminal  Procedure  1  
A)  Elements  and  Components  of  the  Offence  
Reading the Code: basic features of an offence provision: (elements of the offence)
i) substantive: describing the offence
ii) procedural: whether it is an indictable, summary, or hybrid offence
iii) penalty: maximum or minimum

Substantive Parts of the Offence


The substantive part describes the actions or omissions that make up the offence.
-Together described as the “elements of the offence”
-Proven through oral or physical evidence.

A finding of guilt requires Crown to prove, BARD, two general components:


i) Objective component: Actus Reus – the physical act or omission making up the
offence

13
ii) Subjective component: Mens Rea – the accused’s state of mind or level of intention
in committing the act.

Mens Rea - Why is a mens rea element generally necessary in criminal offences?
We generally seek to criminalize only those who:
• Made a choice
• Voluntary
• Know it was wrong, not legally (ignorance of the law is not an excuse).

Different levels of Mens Rea


Levels of intent vary throughout Code from “direct” (or “specific”) intent or knowledge to
negligence

Lesser Forms of Mens Rea:


• recklessness: being aware of potentially harmful consequences, but taking risk
unjustifiably
• willful blindness: refusing to consider whether a risk exists, or ignoring potential
criminal consequences.
• carelessness: failing to appreciate the risk that a reasonable person would have
appreciated.
• criminal negligence: differs from ordinary civil negligence by involving reckless
disregard for safety of others.

B)  Charge  Approval  Standards  


After reviewing the Police and/Witness Evidence in a criminal investigation, Crown must
decide whether to proceed with a prosecution (i.e., have an Information sworn or an
Indictment laid) by asking two questions:
• Given the evidence available is there a substantial likelihood of conviction?
• Is it in the public interest to proceed?

C)  Informations  and  Indictments  


If they decide to proceed they would move onto having an information sworn or an
indictment laid.

Information: the document in which an accused person is formally charged with a


specified offence (or set of them), when the prosecution will proceed in (or begin) in
Provincial Court.
Indictment: the document in which an accused person is formally charged with a
specified offence (or set of them), when the prosecution proceeds in Superior Court.

• When Crown/Police are ready to proceed, a police or witness acts as the


“informant” before a judge or justice of the peace, swearing that they have
reasonable grounds to believe that specified crime has occurred.
• Once signed by Judge or Justice of the Peace, prosecution formally begins and
suspect becomes an “accused” person.

14
• In “Information” - parties, date, jurisdiction, offence/source, the conduct --> the
crown has to prove all of these things beyond a reasonable doubt.

Note: RCC report to crown council from police the crown uses to build their case. A copy
is given to the Crown and to the Accused but not to the Judge.

6.  Introduction  to  Due  Process    


A)  Police  Powers    
a) Major growth and professionalization of police forces relatively recent – early
19th century UK.
b) Required development of laws that defined and restrained powers of the
police.

Police have “common law powers” and powers found in legislation, i.e., Criminal Code,
Police Act, etc., setting out: when they may arrest, detain, or interrogate, and when they
may search/seize property or people.
The Charter limits scope of powers at common law and under Code and other statutes.

B)  Models  of  the  Criminal  Process  


Crime Control Model Due Process Model
1. Crim justice strives to prevent 1. Criminal justice strives for
crime. fairness.
2. Emphasis on victims’ rights 2. Emphasis on defendants’ rights
3. Police powers should be 3. Police powers should be limited
expanded to make it easier to to avoid oppressive conduct.
investigate and convict. 4. Constitutional rights meant to
4. Legal technicalities seen to ensure fairness and consistency,
hinder police and should be not just technicality.
minimized. 5. Objective of crim justice: to
5. Objective of crim justice is to discover legal, not factual guilt.
discover factual guilt.
6. Individual rights important but
not really

• Due process not primarily concerned w/avoiding wrongful conviction or obtaining


reliable evidence.
• Due process rights are meant to preserve core human rights against cruel, inhumane
treatment, and right not to be deprived of life or liberty except through a fair process
that respects a person’s dignity and equality under law.

C)  The  Charter  
• The legal rights (ss. 7 – 14) in the Charter meant to entrench due process and core
human rights (life, liberty, security of the person) as constitutional.
• Marks a break w/earlier emphasis on crime control model.

15
• First 10-15 years after 1982, SCC in strong support of rights; period since, more
restrained.

7.  The  Law  of  Search  and  Seizure  


A)  Origins  
• Common law has long protected against unlawful searches and seizures. In the
past it was based on the right in the tort of trespass. Today deeper association
w/right of privacy
• Has generally required express legal authority (usually a warrant on probable
cause to believe a crime had been committed and that evidence would be found in
place to be searched).

B)  Sources  of  Search  Power  


• In Canada, legal authority for searches/seizures must be found statute or common
law.
• Many statutes (including Code) allow for search to be conducted w/warrants; and
in a few cases (exigent circumstances; reg offences) w/o them.
• The common law sets out a further series of contexts in which police may carry
out warrantless but limited search.
• All searches in Canadian law must now meet standards set out in s. 8 of Charter
Statutory Authorization:
• Many statutes provide for the issuance of warrants in prescribed
circumstances.
• The general provisions of the Criminal Code to deal with searches are
sections for each 487, 488, and 489.
• PO must swear “information to obtain”; and then JP decides to issue a
warrant or not:
Common law search power:
(1) Searches incident to a lawful arrest [Cloutier]
(2) Frisk search as part of a “investigative detention” [Mann]
(3) Enter and search dwelling if in lawful hot pursuit of a suspect
(4) Enter and search dwelling to preserve life or prevent injury [Godoy]
(5) Enter and search a dwelling to make an arrest incident to an arrest warrant
[see e.g., Landry as modified by Feeney], and
(6) Power to conduct a warrantless sniffer dog search in a public place based on
reasonable suspicion [Kang-Brown and A.M.]

C)  Challenging  Searches    
All search powers and procedures; statutory or common law, must comply w/Charter.
S. 8 guarantees: Everyone the right to be secure against unreasonable search or
seizures.
In Collins, the Court held that for a search to be reasonable, it had to be:
1. Authorized by law (statutory or common law);

16
2. The authorizing law has to be reasonable; and (the answer to this question
comes from Hunter)
3. The authorized search has to be carried out reasonably.

• The starting point for any challenge under s.8 of an unreasonable search is REP?
• If no REP then search will always be valid.
• If there is a REP then there can be a s.8 breach. After that, move to step 1 of the
Collins test and, in the situation where the search was authorized by a statute via a
search warrant you have to make sure all the rules of the statute and warrant were
followed:
Challenging Warranted Searches:
1. Were all the procedures for issuing the warrant followed?
• s. 487(1) – outlines procedures for issuing a warrant
1. Has the warrant been properly executed?
• s.29 - P.O. must have warrant and must produce it
• s.488 - search warrants shall be executed by day
• common law [Eccles v. Bourk]
- to announce his/her presence [i.e. knock on door]
- to identify him/herself [i.e. “I am a police officer”]
- give reason [i.e. we have a warrant to search…]
- request entry
• Was the amount of force used reasonable?
• s. 25
• If the Crown fails on any of these three grounds: (and there is no other authority (i.e.
common law) for the search), s. 8 is violated; analysis shifts to 24(2) however, you
can also challenge the statute authorizing the search/warrant:

Challenging the Law authorizing the warrant/search:


Hunter Test (reasonable Warrant test)
Search laws that have:
1. A form of prior authorization (if feasible);
2. A requirement for an independent and impartial person who authorizes the
search; and
3. A sufficiently stringent justificatory standard: ‘reasonable and probable
grounds…’ (to believe that an offence has been committed and that evidence will be
found in that place)
i) Hunter v. Southam Inc. Supreme Court of Canada [1984] SCC
Ratio: In order for a law authorizing a search to be consistent with s.8 of the Charter
(i.e. reasonable) it must have a form of prior authorization (if feasible), have the
requirement for an independent and impartial person who authorizes the search;
and have a sufficiently stringent justificatory standard: ‘reasonable and probable
grounds’...(to believe that an offence has been committed and that evidence will be
found in that place)

17
Facts: The gov’t, under the Combines Investigation Act, began an investigation into
Southam Newspaper. The investigators entered Southam's offices in Edmonton examine
documents. The Act allowed the Director of Investigation to enter premises, examine or
take away materials on the basis of the Directors belief that the evidence was relevant to
the an investigation. Section s.10(3) stated that the director could obtain a certificate of
authorization from a member of the Restrictive Trades Practices Commission.

Issue: Are s.10(1) and 10(3) of the Combines Investigation Act consistent with the s.8
Charter rights concerning search and seizure?

Conclusion: The SCC took a “purposive” approach; what was s. 8 meant to do?
Concluded it was meant to protect more than property. They held that the s. 8 guarantee
against unreasonable search/seizure was designed to protect a right of privacy (i.e. the
right to be left alone by the State) and it should provide citizens with a reasonable
expectation of privacy from government intrusion.

Note: W/o warrant, or not required the presumption that s.8 will be violated can be
rebutted; ie) searches under exigent circumstances, searches of travelers at borders,
administrative and regulatory searches, searches by school officials for contra, band dog-
sniffing in public, searches incidental to arrest, etc.

If the law is deemed reasonable and (as per step3 of the Collins test) the search itself is
deemed reasonable (step 3 Collins test), there are no more issues. However, if the law is
deems unreasonable it may be able to be saved via s.1 of the Charter (see Oakes Test)

Challenging warrantless searches


A search without a warrant, or w/ reasonable and probable grounds, will “presumptively”
violate s. 8 of the Charter but the search may be reasonable if it can pass the Collins Test
(this goes for searches that had warrants but the rules may not have been followed or the
law may have been found to be unreasonable)
1. analysis of validity of search:
1. did person have REP in thing or place?
2. have the conditions in Collins been met?
a. was there common law authority for this?
b. was that authority reasonable? (Hunter Test)
c. was the search carried out in a reasonable manner?

D)  Reasonable  Expectation  of  Privacy    


Note: no clear test in SCC decisions. The case law is complex; lawyers look to decided
cases on point.
What kind of privacy interest is engaged:
• Personal privacy (bodily substances, Stillman)
• Territorial privacy (hotel suite or home, Wong)
• Informational privacy (electricity records or heat signature, Plant, Tessling)
Consider the totality of circumstances: (Edwards, Tessling)
• What was the subject matter of search or seizure?

18
• Does the claimant have a direct interest in the subject matter?
• Does the claimant have a subjective expectation of privacy in the subject matter?
• If so, was the expectation objectively reasonable?
Examples:
• Buhay: REP in rented locker at a bus depot.
• Stillman: REP in bodily samples
• Harrison: REP in vehicle.
• Edwards: no REP in his girlfriend’s apartment (territorial)
• Belnavis: passenger has no REP in bags in the passenger portion of the vehicle.
• Tessling: no REP in heat pattern emanating from home.
• Patrick: no REP in garbage at curb
-no clear line/test in case law… Yet some principles, or bright lines, do emerge…
i) R. v. Edwards Supreme Court of Canada [1996] SCC
Ratio: Whether or not a REP attaches will depend on the “totality of the
circumstances.”

Facts: Edwards was convicted of possession of drugs for purposes of trafficking. Two
officers went to his girlfriend's apartment and gained entrance. Once inside, the accused's
girlfriend directed them to the location of a significant cache of drugs. The accused's
appeal was dismissed w/a dissenting opinion that found a REP giving rise to the
possibility of an infringement of his s. 8 Charter rights against unreasonable search or
seizure.

Issue: Does the accused have a REP in his girlfriend’s apartment and if so, was the search
conducted there a violation of his s.8 Charter rights?

Conclusion: The court held that despite the fact that he had keys, he had no REP because
he didn’t have full control over access; was a “privileged guest.” They outlined a 7 part
test looking at the ‘totality of the circumstances’
• presence at time of search
• possession or control of the property or place searched
• ownership
• ability regulate access
• historical use
• subjective expectation to privacy
• objectively reasonable
ii)R. v. Wong Supreme Court of Canada [1990] SCC
Ratio: Individuals have a REP in their interactions from the state when state
surveillance is involved.

Facts: Accused charged w/keeping a common gaming house. Crown sought to do video
surveillance of the hotel room. The video was obtained surreptitiously.

19
Issue: By indiscriminately inviting strangers into his hotel room, did Wong still have a
REP?

Conclusion: The court found that although by allowing others into his room, Wong could
not have a REP from the others he invited in, he still would have a REP against the state.
The accused rights under s.8 had been violated because no warrant was obtained for video
taping however, the court held that the admission would not bring the administration of
justice into disrepute under s.24(2) b/c the police acted in good faith and there was no
Criminal Code provision for them to get warrant so the videotape was admitted.
iii) R. v. Plant Supreme Court of Canada, [1993] SCC
Ratio: Individuals may have a REP over information deemed to be part of their
biographical core of personal information.

Facts: Police accessing hydro-consumption records for a residence w/o a warrant.

Issue: Did the individual have a REP over his hydro-consumption records and if so was
the search of those records a violation of his s.8 Charter rights?

Conclusion: The court recognized that s.8 protects a “biographical core of personal
information which individuals in a free and democratic society would wish to maintain
and control from dissemination to the state. This would include information which tends
to reveal intimate details and personal choices of the individual.” The court outlined
several questions to determine if information was part of the core:
• The nature of the information
• The nature of the relationship b/w the party releasing the info and the party
asserting confidentiality
• the place where the information was obtained
• the manner in which the information was obtained
• the seriousness of the crime being investigate.
The court found that the information didn’t touch on the biographical core of the accused
and therefore he had no REP in his hydro consumption records. Also, it was available to
the public.
iv) R. v. Tessling [2004] SCC
Ratio: Whether there exist a REP in a given situation depends on whether someone
has a subjective expectation of privacy and whether the expectation is objectively
reasonable having regard to totality of the circumstances, biographical core.

Facts: The police used a thermal imaging device to take a “heat” picture of the
respondent’s home while flying over the house. They did this w/o a warrant.
The image, along w/other evidence, caused the police to infer the existence of a grow-op.
The respondent says the use of the FLIR tech constituted a search of his home and having
been done w/o a warrant violated his s.8 rights.

Issue: Did Tessling have a REP in the heat patterns detectable from exterior of house?

20
Conclusion: The Court held that there was no REP in the heat pattern as it did not touch
on the biographical core nor, in the totality of the circumstances, would a ROP expect to
have privacy over heat patterns of their house. The court was careful to qualify that this
decision was concerning this technology in its current capacity. Overall the court held that
whether there is REP depends on whether:
a. someone has a subjective expectation of privacy
b. whether expectation is objectively reasonable having regard to totality of
the circumstances.
Court thus blends Plant and Edwards factors together, and also considers intrusiveness of
search.
v) R v Patrick, Supreme Court of Canada [2009] SCC
Ratio: There is no REP in individual’s trash that is on the curb for pick up.

Facts: Acc’d suspected of running a meth lab; police search garbage placed at curb, but
within property line.

Issue: Do we have a REP in our garbage at the curb?

Conclusion: The courts held we do not have a REP in our garbage if it is placed on the
curb, maybe closer to the house?

8.  Arrest  
A)  Police  Interactions  with  Citizens  
a) Powers of arrest governed by the Code, the common law and the Charter.
b) Code provides for powers of arrest both with and without a warrant.
c) as with Search and Seizure, these laws and rules are aimed at striking a balance
between state’s interest in prosecution and individual liberty.
d) Governing principle: intrusions on liberty should be minimal.
Police Questioning
• Police are entitled to ask citizens questions (e.g., name, where do you live, etc.).
• Citizens have no legal obligation to cooperate and can refuse to cooperate.
• If a person does not cooperate, a police officer may not grab or forcibly detain
them unless they have lawful authority to do so.
Investigative Detention
• Somewhat in between
• Police have power, recently recognized at common law, to detain briefly for
investigative purposes – based on ‘articulable cause’ or ‘reasonable
suspicion’: Simpson, Mann, Clayton, Grant, Suberu
What is an Arrest?
• An arrest is a process by which a police officer or other person is authorized by
law to compel person to appear before a judicial officer of the Court to answer a
charge.

21
• An arrest consists of words of arrest accompanied by a physical detention of the
suspect; or by implicit or explicit threat of physical detention that causes the
suspect to submit.
• An arrest is only one method – the most intrusive - that can be used to compel an
accused’s appearance before a Court.
• Other, less intrusive, methods: Appearance notice, a summons,
• all require judicial confirmation

Written Request Physical Compulsion


Judicial confirmation Summons (to be served) Arrest with a warrant
before
Judicial confirmation Appearance notice Arrest w/o a warrant
after (caught doing something
really serious)

B)  Arrest  without  Warrant  


• Governed by 494 and 495 of Code
• Two main ways Code distinguishes different arrest powers:
1. “Indictable” versus “criminal offences”; and
2. “finds committing” versus “reasonable grounds”
1) Citizens Arrest
• Section 494 (a) and (b) of the Code authorizes “anyone” to arrest a person who he
or she:
a. finds actually committing an “indictable offence”; or
b. has reasonable grounds to believe has committed a “criminal offence”
and is escaping from and is being freshly pursued by someone with lawful
authority to arrest the person. (note have to pass over to police quickly)
1. Section 494(2) creates a broader power for property owners to arrest someone
actually committing a “criminal offence” in relation to property.

2) Arrest by Police Officer


Section 495 confers wider powers of arrest on P.O. A P.O. may arrest without warrant in
three situations:
• With reasonable grounds to believe a person has committed or is about to commit
an “indictable offence”;
• A person the officer finds committing any “criminal offence”; or
• A person who officer has reasonable grounds to believe is the subject to an arrest
or committal warrant.

i) R v. Storrey [1990] SCC


Rule: S.450(1) requires that an arresting officer must subjectively have reasonable
and probable grounds on which to base the arrest. Those grounds must, also be
justifiable from the perspective of an ROP who would conclude there were
reasonable and probable grounds for the arrest.

22
Cory J: “the Criminal Code requires that an arresting officer must subjectively have
reasonable and probable grounds on which to base the arrest. Those grounds must, in
addition, be justifiable from an objective point of view. That is to say, a reasonable
person placed in the position of the officer must be able to conclude that there were
indeed reasonable and probable grounds for the arrest.”
-subjective ‘reasonable and probable grounds’ on part of police officer are not enough

3) Catch and release


• Part XVI exhibits a preference to release rather than arrest.
• Provides multiple opportunities to do so including after initial arrest.

But a P.O must not arrest…


• 495(2) states that officer shall not arrest if it is
o a less serious offence (summary, hybrid or s. 553 offence);
o the officer has reasonable grounds to believe arrest is not necessary in the
public interest; and
o officer has no reasonable grounds to believe person won’t attend court.

Evaluating public interest involves considering:


1. Ascertain ID
2. Preserve Evidence
3. Flight Risk
4. Prevent you from continuing the offence (or the offences)
5. Safety of the accused or of others
• PO may then release on appearance notice or summons

Release by ‘Officer in Charge’


1. Where P.O. arrests [i.e., does not release], s. 497 provides that Officer in Charge
of the police station “shall as soon as practicable” release the accused if the person
is charged with offences in 496(a),(b), and (c), or offences with jeopardy less than
5 years, on:
i. An appearance notice; (can attached conditions)
ii. A promise to appear; or
iii. A recognizance without sureties or deposit,
unless it is not in the public interest to do so.
If not released, person will appear before a justice usually within 24h.

C)  Arrest  with  Warrant    


1) S. 504 permits anyone who has reasonable grounds to believe an indictable
offence has been committed to lay an information under oath before a J.P.
2) Section 507 of the Code provides that a J.P. shall receive such information and:
where the JP considers a case has been made out to compel an accused to
answer a charge, shall issue a summons unless there are reasonable
grounds to believe it is necessary to issue a warrant for the arrest of an
accused.
• Endorsed warrant: PO may release A on PTA or recognizance with conditions.

23
• Unendorsed warrant: PO usually brings A into custody for bail hearing.

9.  Ancillary  Powers:  Incident  to  Arrest  and  Power  of  Detention    


1) Courts have, under the common law, recognized that police have some powers
that are “ancillary” to a lawful arrest.
2) However, the lawfulness of the exercise of “ancillary powers” is conditioned by
the lawfulness of the arrest.

A)  Ancillary  Powers  Incident  to  Arrest  


i) Cloutier v. Langlois [1990] SCC
Ratio: In cases where police have grounds for an arrest, they do not need additional
grounds to carry out a frisk search. The scope of a search incident to arrest is not
limited to “frisk” but can include “surrounding area” so long as it is for the short-
term purposes to find items that might harm officer or help escape or to prevent
evidence from being destroyed. (Frisk Search Incident to Arrest)

Facts: Cloutier was stopped by police while driving. They learnt that he had an
outstanding warrant. The constables informed him and asked him to accompany them to
the police station. When they asked the respondent to get into the patrol car, the
constables carried out a "frisk" search: the hands of the accused were placed on the hood
of the car, his legs spread and the constables patted him down. The respondent was then
taken to the police station.

Issue: If the police have grounds for an arrest, do they also (automatically) have the
authority to carry out a pat-down or ‘frisk’ search? Or do they need additional ‘RPG’ to
search or is reasonable suspicion enough?

Conclusion: The court held that they do not need additional grounds for “search incident
to arrest” (i.e., don’t need separate RPG to believe weapon/evidence will be found on
person.) And it was noted that:
1. This power does not impose a duty, but creates a discretion.
2. The search must be for a valid ‘criminal justice’ objective, such as officer safety
or evidence related to offence. It cannot be used to intimidate or pressure the
accused.
3. The search must not be conducted in an abusive fashion

It was also held that the scope of a search incident to arrest was not limited to “frisk” but
can include “surrounding area” so long as it is for the short-term purposes:
1. to find items that might harm officer or help escape;
2. to prevent evidence from being destroyed.

NOTE Exception
a) Stillman (1997): “search incident to arrest” does not extend to seizing bodily samples.
It thus rejects – for search of a person – expansion of purposes of “search incident to

24
arrest” include: preventing evidence from being destroyed [teeth impressions won’t
change!], and gathering evidence relevant to guilt/ innocence
ii) R v. Golden [2001] SCC
Ratio:The automatic authority to carry out a search incident to arrest doesn’t
extend to strip searches. For a strip search to be warranted, there must be additional
Reasonable and Probable Grounds to believe a strip search is necessary to discover
weapons or evidence.

Facts: Police officers observed Golden giving what they believed was cocaine to people.
During the arrest, police found what they believed to be crack cocaine under the table
where one of the suspects was arrested and Golden was observed crushing what appeared
to be crack cocaine between his fingers. After the arrest, the Police officers performed a
very public strip search and eventually located drugs between Golden’s buttocks.

Issue: Does the automatic authority to carry out a ‘search incident to (lawful) arrest’
extend to strip search searches? And if so can these still be reasonable if carried out in
public?

Conclusion: Courts first distinguished b/w frisk and strip searches; strip searches are
inherently degrading and a serious invasion of privacy. Thus, there should be no
automatic authority to do this and are only permitted where there are “reasonable and
probable grounds” to believe a strip search is necessary to discover weapons or evidence
related to the offence for which the accused is arrested. Also it must be done in
“reasonable” manner (at Police station, unless exigent circumstances). Therefore the
search at the restaurant in this case was not reasonable and the evidence was excluded
under s.24.
iii) R. v. Caslake,[1998] SCC
Ratio: When relying on the power of search incident to arrest, the search must truly
be related to the arrest in order to be valid.

Facts: An RCMP officer, several hours after arresting the accused for possession of
narcotics, conducted an inventory search of the accused’s impounded car pursuant to
police policy and found cash and two individual packages of cocaine. He did not have
permission or a search warrant.

Issue: Whether the search of the car was consistent with s. 8 which guarantees the right to
be secure against unreasonable search or seizure, and if not consistent, whether the
evidence should have been admitted?

Conclusion: Stillman/Collins analysis (pre-Grant) to arrive to the conclusion that the


evidence would not be excluded despite the search not being reasonable.

Where the Crown relies on doctrine of ‘search incident to arrest’,.. “the police must be
able to explain, within the purposes recognized in the jurisprudence (protecting the
police, protecting the evidence, discovering evidence) or by reference to some other

25
valid purpose, why they conducted a search. They do not need reasonable and
probable grounds. However, they must have subjectively had some reason related to
the arrest for conducting the search at the time the search was carried out, and that reason
must be objectively reasonable. Delay and distance do not automatically preclude a search
from being incidental to arrest, but they may cause the court to draw a negative
inference. That inference may be rebutted by a proper explanation.” In this case, the
reason for searching it was inventory and thus was not truly related to the arrest but the
delay was no issue.
A police search of the car for the purpose of finding evidence which could be used
at the accused’s trial on the charge of possessing marijuana for purposes of trafficking
would have been well within the scope of the search incident to arrest power. However,
the police cannot rely on the fact that, objectively, a legitimate purpose for the search
existed when that is not the purpose for which they searched.

Summary of Search Incident to Arrest


There are thus two separate tests:
1) For a person: officer can search accused for purpose of protecting themselves,
or for preventing escape, and in every case, for evidence that may be
destroyed.
2) But for a vehicle (or other location): s/he can go further and also search for
evidence not in danger of destruction.

• police have power in common law to enter dwelling house w/o warrant only if in
“hot pursuit” of suspect – otherwise, warrant is necessary to enter for purpose
of arrest. OR
• s. 529.3 of Code: power to enter dwelling house w/o warrant for exigent
circumstances (imminent bodily harm or death; imminent loss of evidence.)
• s. 489: in the course of executing a warrant, police may seize anything they have
reason to believe was obtained by commission of an offence – even if it’s not
mentioned in warrant.
• plain view doctrine (common law): where police have grounds to detain/arrest,
they may seize anything unlawful in plain view (even if it does not pertain to
reason for arrest).

B)  Powers  of  Detention    


• Prior to R v. Simpson and R v. Mann, police had no common law power to detain a
person for investigative questioning. There was no in-between, arrest or not.
i) R. v. Simpson, 1993 (ON CA)
Ratio: Short of arresting a person, Police can stop and detain a person briefly for
questioning if they have a reasonable suspicion that the detainee is criminally
implicated in the activity under investigation.

Facts: The police officer was informed of a local house that was thought to be a ‘crack’
house. While paroling the area the officer stopped Simpson in a vehicle. While
questioning him, he noticed a bulge in his pocket. The officer, after feeling the hard lump,
asked Simpson to remove it. Simpson attempted to throw it away and the Officer stopped

26
him and forced a bag of cocaine out of his hand. The officer stated after that he did not
have RPG to arrest until he saw that Simpson had cocaine on him.

Issues: Do police have a common law authority to detain a person where they do not have
authority to arrest? (remember arrest, rpg needed that an offence was perp by the
individual) Was the stop/detention and subsequent search lawful? Should the cocaine be
admitted?

Conclusion: Court held that where a particular individual is stopped and detained by
police for questioning to determine if he/she is involved in a specific offence, that
detention can be justified, but only if the detaining officer acts on a “reasonable
suspicion” or has “articulable cause” to this effect. (Lower standard the RPG) It was
stated that “articulable cause/reasonable suspicion” is assessed on an objective standard
that is beyond a hunch or intuition gained by experience; but lower standard than
reasonable and probable grounds to arrest or search.

General Notes:
• first recognition of a police power to detain individuals for investigative purposes
without having to arrest them.
• An arrest must be based on reasonable and probable grounds that an offence has
been committed.
• This was a large shift and fundamentally changed the legal relationship between
police investigations and the individual liberty interest of the general public.
ii) R. v. Mann, [2004] SCC
Ratio: Where an officer has reasonable grounds to believe that his or her safety, or
the safety of others, is at risk, he/she may perform a search incident to detention.

Facts: Officers were responding to a break and enter when while searching the
neighborhood, they spotted a young man matching the description of the suspect; a 21
year-old, 5 foot 8, Aboriginal male in a black jacket. The officers stopped the man, asked
him some questions, and then gave him a pat-down. When patting the man down, the
officer noticed a soft object in one of his pockets. The officer reached in and pulled out a
bag containing 27 grams of marijuana.

Issue: Whether detention and search of accused was lawful under ss. 8 and 9 of the
Charter; and if not, whether evidence should be excluded?

Conclusion:
The court held that:
0) Police have a power of investigative detention; and
1) Police may carry out limited search incident to lawful detention (more limited
than search to arrest). Only where officer has reasonable grounds to believe
that his or her safety, or the safety of others, is at risk.

The SCC substantially adopts the approach taken in Simpson. Police officers may detain
an individual for investigative purposes if:

27
• there are reasonable grounds to suspect in all the circumstances that the individual
is connected to a particular crime; and
• such a detention is necessary in light of the totality of the circumstances in order to
determine whether the individual is involved in a particular crime.

Also Held that investigative detention should be brief in duration and doesn’t impose an
obligation on the detained individual to answer questions. Searches can’t be justified on
basis of vague or nonexistent concern for safety, or on hunches or personal intuition. In
this case, the detention was lawful however the evidence was excluded because there were
no reasonable grounds to believe that the police officers safety was at risk from a soft
bulge.
iii) R. v. Clayton, [2007] SCC
Ratio: Police have broad, general common law power to stop/detain for investigative
purposes [i.e., not just roadblocks] when it is reasonably necessary based on “the
totality of the circumstances.”

Facts: Police were called to a shooting outside a strip club. They blocked the exits to the
parking lot. They stopped a car and found two guns, one after the suspect tried to escape.

Issue Do police have a general detention power at common law such that they can
establish a roadblock for investigative purposes?

Conclusion: The court held that police have broad, general common law power to stop
for investigative purposes [i.e., not just roadblocks] when it is reasonably necessary based
on “the totality of the circumstances”, which includes:
1) the nature of the situation (including seriousness of the offence),
2) the information known to the police about the crime and/or suspects,
3) whether the stop/detention (i.e. geographic and temporal scope) is reasonably
tailored to the offence, and
4) in balancing the risk to the public versus the liberty of citizens, the stop is no more
intrusive than “reasonably necessary”.

C)  Detainment  
When is a person detained?
1. Before the police can detain anyone they must have reasonable and probable
grounds to believe that an offence has been committed. Then if they want to
search the search must be conducted in connection with an arrest as shown in;
a. Mann shows us that the reasonable suspicion of the police must connect a
specific individual to a specific offence.
b. Clayton, investigation can be carried out when reasonably necessary in the
circumstances.
2. However there were still questions about what s. 10(a) and (b) rights subjects of
investigative detentions have, and when exactly detainment occurs.

28
i) R v Grant [2009] SCC
Rule: Psychological detainment will be found when a ROP in the detainee’s
circumstances would conclude that they have been deprived of the liberty of choice
and must therefore comply with the police. Three factors will be assessed; the
circumstances giving rise the encounter, the nature of police conduct and the
characteristics or circumstances of the detainee.

Facts: Same as Grant facts from above, intimidated by 3 officers.

Issue: 1) Was Grant detained within the meaning of s.9?


2) Was there reasonable suspicion connecting Grant to an offence?

Analysis: Courts defined detention as physical or psychological. Psychological


detainment being a situation where a reasonable person either has or feels they have a
legal obligation to comply. In cases where there is no physical restraint or legal obligation
the court looks objectively at these three factors to determine if someone is detained;
i. The circumstances giving rise to the encounter; as they would be reasonably
perceived: Whether the police were providing general assistance,
maintaining general order, making general inquiries regarding a particular
occurrence, or singling out the individual for focused investigation.
ii. The nature of the police conduct; language used, use of physical conduct,
place where the interaction occurred, presence of others, duration of the
encounter.
iii. The particular characteristics or circumstances of the individual; age,
physical stature minority stats, level of sophistication.

The Court finds initially he wasn’t detained however once is he instructed to “keep his
hands in front of him” he was detained. Using the test above they examine, the singling
out of Grant, the intimidating stance of the officers, and the fact that Grant was a younger
man, as factors that make it reasonable to perceive he was detained.

Conclusion: The court found that because the officers had no legal grounds for
detainment his s.9 rights were violated, and because he was detained his s.10(a) and (b)
rights were also violated.
ii) R v Suberu, [2009] SCC
Rule: When police detain someone they must;
§ Immediately advise the detainee of their s.10(a) and (b) right’s (a: reason for
detention, b: to retain and instruct counsel)
§ Refrain from eliciting incriminatory evidence until the detainee has had
reasonable opportunity to reach a lawyer or have unequivocally waived this
right.

Facts: Suberu and a friend used a stolen credit card to buy gift cards for Walmart and
LCBO’s. They went in a town and Suberu’s friend tried to use a card, the store was aware
and called the police. Upon arriving Suberu tried to sneak out saying “I guess you won’t

29
need me” officer goes after him and questions him while he is sitting in his van, until he
has enough information to arrest him he reads him his s.10(a) and (b) rights.

Issue: 1) Was Suberu detained (when he was talking to Roughley while sitting in his van)
within the meaning of the Charter?
2) If he was detained was he entitled to be told his s.10(b) rights as soon as
detention began?

Analysis: The court goes through the 3-point test from Grant:
• The circumstances giving rise to arrest; were preliminary, police were orientating
themselves with the investigation, and Suberu attracted attention with his
statement.
• Nature of police conduct: a reasonable person wouldn’t have thought that Suberu
was being detained.
• Individual’s personal circumstances: using ROP there is no evidence to suggest that
Suberu didn’t think he could leave. Suberu also never asserted that he didn’t want
to speak.

Conclusion: The officer was only engaged in preliminary questioning, and not detained
so his s.10(b) rights were not violated. The appeal is dismissed.

Criticisms of the test of detainment from Grant:


• It isn’t a clear test, police don’t have straightforward rules and therefore has
potential to lead to more detentions of minorities because of their background and
that they feel unable to disregard police directions.
• There is also a lack of court review on this matter because the court has no
remedy.
• Also because of the police requirement to give s.10(a) and (b) rights it is likely the
courts will work to say a person isn’t detained until they receive these rights.
• Possible solution would be a clearer test, not have s.10(b) rights during
investigative detention or for police to make it explicit when there is no
detainment.

10.  Confession  
Common Law Confession Charter Protections
Who is bound? A “person in authority” An “agent of the state,”
determined on a subjective determined on an objective test
test
When is it triggered? Any time a person makes a Only when the suspect is arrested
statement to a person in or detained
authority
Protection against Yes Yes [s.7]
involuntarily made
statements?
Protection of right to No Yes [S.10(b)]

30
counsel and to be
informed of this?
Protection of a post- Applies both before and Only applies after detention
detention “right to after detention (but right to [s.7]
silence”? silence not protected against (Hebert/Singh)
undercover officer:
Rothman)
Burden and Standard Crown, beyond a reasonable Applicant (accused) balance of
of Proof doubt probabilities at both the breach
and exclusion of evidence stages
Effect of Breach Automatic Exclusion Must go through s.24(2) analysis

A)  Admissibility  of  Confessions  


• Admissibility of confessions comes from the common law not statute
• A confession is an acknowledgement or admission of a legally relevant fact.
• The common law has held that evidence that is relevant is admissible even if
obtained illegally, through the use of a special rule:
o A confessions isn’t admissible unless the Crown can prove BARD it is
voluntary.
o If a reasonable doubt about voluntariness is raised it is excluded, if they
can’t prove BARD voluntariness it is excluded.
• the admissibility of confessions can be challenged through the common law or
under s.7 or s.10(b).
i) Boudreau v. The King, SCC [1949] S.C.C
Rule: Affirms Ibrahim v. The Queen. Comprised of two points:
• This rule only applies to statements made to persons in authority (PIA)
• This statements will only be rendered inadmissible if the PIA made threats or
promises (inducements)

Analysis: They were concerned with confessions that were instigated or coerced, the
overall reliability and truth of the confession. They were worried inducements
/intimidation might influence subjects to make false confessions. Statements made to PIA
must have occurred without fear of prejudice or hope of advantage.

Person in Authority: Someone engaged in law enforcement, or in the arrest, detention,


interrogation or prosecution of the accused.
• If a statement was made to a PIA then based on the Boudrea/Ibrahim rule the
Crown had to show the accused made the statement w/o fear of prejudice or hope
of advantage.
ii) R v Wray SCC [1971]
Facts: The police extracted a confession using threats and inducements (contrary to
Boudreau it was involuntary). The accused admitted he killed her and tells police where
he threw the gun, which they found.

31
Holding: SCC held the gun was admissible, despite being obtained from an involuntary
confession, as well as parts of his confession that led to the discovery of the gun b/c they
were considered reliable b/c the gun was found. However they excluded the statement “I
killed her” and parts not confirmed by discovery of the gun.

Post Boudreau:
• Courts recognized circumstances where an involuntary statement should be
excluded even though there was no inducement or prejudice.
• Oppressive conduct (Hobbins [1982])
• Operating mind (R v Clarkson [1986])
• Shocking Police Conduct (Rothman v The Queen [1981])

iii) R v Whittle [1994]


Holding: operating mind requirement, court applies a restrictive test; accused only needs
sufficient cognitive capacity to understand what his is saying and what is said to him.”
iv) Rothman v The Queen, SCC [1981] SCC
Rule: Test of the admissibility of statement made to a non-PIA; it is inadmissible if
there is inducement/treats by the non-PIA which would cause the statement to be
untrue and that would bring the administration into disrepute, it is admissible if it
was elicited in circumstances that would not render it inadmissible.

Facts: Rothman was arrested for possession of hash with intent to traffic. He was asked
if he was willing to make a statement but declined. He was then put in a cell in which
there was an undercover officer dressed like a criminal. They conversed and Rothman
made incriminating statements.

Issue: Whether the statement was obtained by a PIA is subjective not objective based on
questions to the accused. Who did Rothman think he was speaking to?

Analysis: Through a subjective test the court had to determine whether Rothman truly
believed that the police officer was a PIA.

Conclusion: The court found his admission could be admitted because it wasn’t made to
a person in authority and was therefore voluntary. The appeal was dismissed.

v) R v Grandinetti; re-affirmed the decision in Rothman that an undercover officer is not


usually viewed by the accused as a PIA, and generally speaking undercover officers wont
be considered PIA’s.

B)  Evaluating  voluntariness  
Inducements:
• ‘Quid pro quo’ is one possible form of inducement. Though it is not always, and it
only improper when it is strong enough to raise a reasonable doubt.
Oppressive circumstances:
• When the suspect is deprived of

32
o Sleep, water, medical help, presented with fabricated evidence, questioned
aggressively, or questioned for a long period of time.
• Again these must raise a reasonable doubt about the voluntariness of the
confession.

11.  Confessions:  Common  Law  Rules  


i) R. v. Oickle SCC [2000]
Rule: Common Law test for Confessions:
• Confession should be excluded if police deception shocks the community.
• If not rising to this level, carry out “overall voluntariness analysis” (which
will include deception as a factor).
The Crown must show that the confession:
o It wasn’t over borne by threats or inducements, oppressive
circumstances, deception.
o It is not due to the lack of an operating mind.

Facts: Oickle is an arsonist who was brought in to do a lie detector test. They said that
they would make his girlfriend do a polygraph test if he didn’t confess, and intimated that
they would be able to get him psychiatric help. At trial they found his confession
voluntary, CA then found that it was involuntary.

Issue: Did the CA err (voluntariness is a question of fact only overturned if there is
palpable error by the trial judge) in finding his confession to be involuntary?

Analysis/Conclusion: The court found Oickle wasn’t induced the questioning wasn’t
hostile and the inducements didn’t lead to the belief that there would be fewer legal
consequences. The trial judge didn’t make a palpable error, statement was voluntary.
J. Arbour dissented that the statements were induced by promises made, and it wasn’t
voluntary based on all the actions of the police.
ii) R. v. Spencer SCC [2007]
Rule: Strength of Quid Pro Quo must be assessed and considered in “the overall
contextual analysis into the voluntariness of the accused’s statement.”

Facts: Spencer arrested for multiple robberies, one involving girlfriend. Offers to confess
in exchange for leniency; officer declines. S later requests to see her and is told he can
once he “cleared his plate”.

Issue: Was this a strong enough inducement (“Quid Pro Quo”) to raise a doubt about
voluntariness?

Conclusion: Mere fact that a quid pro quo is found is not determinative. It does not
decide the issue, you can have a weak quid pro quo.

33
12.  Confessions:  The  Right  to  Silence  and  Right  to  Counsel  
• There are two bases on which an accused person might challenge the admissibility
of a confession:
a. Common law confessions rule
b. s. 7 or 10(b) of the Charter
1) If you violate the Common Law rule, evidence is automatically excluded, the
burden here is BARD.
2) If a Charter right is violated you apply s.24(2), the burden here is on a balance.
i) Clarkson v The Queen, SCC [1986]
Rule: Operating mind requirement has 2 elements; comprehension of what is being
said, and comprehension of consequences of making a confession. Test for waiver of
10(b) is whether waiver is clear and unequivocal, must be sufficiently sober.

Holding: The police should delay questioning until the accused is sufficiently sober
to give waiver of s.10(b) rights.

Facts: Clarkson is very drunk found @ home where husband has been shot, there are no
finger prints on the gun. She makes drunken incriminating statements to sister and aunt
after the arrest. She was given 10(b) @ house and station, she is found highly intoxicated.

Issue: What is the test for a valid waiver of 10(b), can an intoxicated person validly waive
their 10(b) rights?

Analysis: The purpose of s.10(b) is the fairness of confessions, less about reliability/fear
of false confessions. They find that the test for waiving s.10(b) is whether the waiver is
clear and unequivocal and the person is in a sufficiently sober state as to be fully aware of
the consequences of their waiving this right.

Conclusion: There was no urgency, they should have waited. Excluded under 24(2) b/c it
was exploitation and would bring police into disrepute.
ii) R. v. Manninen SCC [1987]
Rule: 10(b) imposes 2 duties on police: 1) provide a reasonable opportunity to retain
and instruct counsel w/o delay, 2) refrain from questioning until this reasonable
opportunity has be granted.

Facts: Lawful arrest, police give 10(b). Manninen says “I want to see my lawyer”, police
continue questioning. There was a phone was in room where arrest/questioning was
carried out.

Issue: Once a suspect has requested counsel, what must police do? Can you impliedly
waive 10(b) if you answer questions?

Application/Conclusion: Right to counsel clearly infringed here:


• No reason for delay, police should have let him use phone.

34
• No need for him to ask to use phone, he should have been told he could.
• No implied waiver here just because he responded to questions
• He had right not to be asked further questions (until he could contact counsel).
Police must abstain until then.
24(2) analysis found the breach was serious and deliberate, disregarded under 24(2).
iii) R. v. Brydges SCC (1990)
Rule: Police have a duty under 10(b) to tell suspects of the availability of duty
counsel and legal aid. Also detainee can implicitly waive but the standard is high
(Clarkson; clear and unequivocal).

Facts: Accused was arrested in MB for murder; given 10(b) rights. When taken to
Brandon for questioning, he asked “if they have any free legal aid or anything like that up
here” and says: “won’t be able to afford anyone, heh? That’s the main thing.” Police
Officer asks if there is any reason for wanting to talk to a lawyer and he replies “not right
now no.” He is then questioned, gives incriminating statements, then asks and is provided
counsel.

Issues: Did he implicitly waive right to counsel, by saying not right now no? Do police
have to inform suspects of availability of duty council?

Analysis/Conclusion: Accused was left w/mistaken impression that inability to afford


prevented him from exercising 10(b) right; waiver was therefore not fully informed;
therefore invalid. He had not made an informed choice. Therefore his waiver was
equivocal, he should have been informed and his right was breached, the evidence was
excluded under 24(2).
iv) R. v. Prosper SCC [1994]
Rule: Police have a duty to hold off when detainee asserts right to counsel, but no
duty to provide/pay for counsel.

Facts: Prosper was pulled over for erratic driving. Prosper was arrested for drunk
driving and read his 10(b) rights. He declined to phone a lawyer in private practice,
claiming he could not afford it. Prosper then took a breathalyzer test, and claimed in trial
that the breathalyzer results were taken in violation of right to counsel under 10(b)

Issues: Does the state have an obligation under 10(b) to provide [pay for] duty counsel for
immediate/temporary advice during non-business hours?
If a person can’t reach duty counsel or pay for duty counsel, do police have a duty to hold
off?

Analysis: No duty to provide/pay for it, because this was not intended by drafters of
Charter. As a result pretty much everyone in Canada has 24 hr duty counsel.
v) R. v. Smith SCC [1989]
Rule: Police duties under 10(b) (reasonable opportunity) only apply if accused is
reasonably diligent in exercising 10(b) rights.

35
Facts: Accused arrested at for a robbery and given the number of a lawyer he chooses,
but tells police he won’t call until the morning. He is interrogated an hour later despite
repeated insistence that he doesn’t want to speak until he reaches his lawyer. Eventually
provides inculpatory statement.

Issue: Was 10(b) violated, or was the accused not reasonably diligent in exercising his
right?

Analysis: The accused wasn’t diligent, should have at least asked police to delay in
questioning until morning (he simply declined to even call his lawyer).
Dissent: Finds waiver to have been equivocal, just b/c Smith wanted to wait until morning
doesn’t mean he wasn’t reasonably diligent. There was no urgency.
vi) R. v. Hebert SCC [1990]
Rule: S.7 will be violated if police (undercover officers included) actively elicit
information after their right has been asserted. Police can continue to question.

Facts: Hebert was arrested for robbery, given 10(b); speaks to lawyer, tells police officer
he doesn’t wish to provide statement. Placed in cell w/an undercover officer, he engages
him in conversation and Hebert incriminates himself.

Issue: Did he assert his right to silence? If they solicit his confession by an undercover
officer have they violated his right to silence?

Analysis/Conclusion: (McLachlin) Purpose of right to silence is to respect suspect’s right


to choose whether to speak or not (i.e., respect dignity, autonomy). However here the
police did more than merely converse, they actively elicited the information, thus
subverting Hebert’s right to silence, found serious, excluded under 24(2).
vii) R. v. Singh SCC [2007]
Rule: Police may continue to question a detainee despite repeated invocations of
right to silence as long as they don’t do anything that would render the statement
involuntary. However continued questioning may raise questions about
voluntariness.

Facts: There is a shooting @ club, a bystander hit. A gun is found, not linked to Singh
however he was ID’d by a number of witnesses. At the station he was given 10(b), spoke
to counsel but declined to provide statement. He is still questioned and he invokes his
right to counsel 18 times. Many incriminating statements made. Under the common law
the statements are voluntary; no breach of s. 7 b/c he was aware he was speaking to
someone in authority

Issue: Do police violate s.7 if they continue to question an accused despite repeated
assertions of will to remain silent?

Analysis/Conclusion: A majority held that if a statement is found voluntary at common


law, it is necessarily consistent with s. 7; and vice versa: a finding that s.7 is breached

36
would entail a reasonable doubt about voluntariness. In this case Singh was talkative and
the court found no reason to find his free will was overborne. The rational here was that
because the officer wasn’t undercover the accused was aware they were talking to a PIA.
Minority: If a detainee “firmly and unequivocally” asserts desire to remain silent,
continued questioning violates s. 7. Therefore Singh’s will was overborne.

How does this differ from Miranda rule in US?


1. Miranda rule is that as soon as the “right to silence” is requested, police must
stop questioning
2. In the US there is a right to council at all times.
3. In Canada, unless there is a material change you don’t have the right to
consistently ask for a lawyer again and again.
viii) R. v. Sinclair SCC [2010]
Rule: Accused has no right to further consultation unless there is a change/evolution
in circumstance; new procedures, change in jeopardy, 1st advice is deficient, no right
to counsel throughout, subject to common law voluntariness analysis.

Facts: Sinclair arrested for murder, speaks twice to his lawyer then is questioned for
several hours, and repeatedly invokes right to silence. Asks to speak to his lawyer again,
but is declined. Questioning continues, Sinclair implicates himself in statements made.

Issues: Are police required to give detainee access to counsel beyond initial consultation?
Do you have a 2nd change to speak to a lawyer?
Do you have a right, under 10(b) to have counsel present during interrogation?

Analysis: Majority: No right to further consultation unless change in circumstances


occurs, including:
• New procedures involving detainee (e.g., request to engage in line-up;
polygraph)
• Change in jeopardy
• Reason to believe first advice was deficient/or detainee didn’t understand
consequence of waiver of 10(b)
• No right to have counsel present throughout custodial interrogation. [adults]
But all of this is still subject to common law voluntariness analysis.
Binnie’s Dissent: Detainee should have right to further consultations where
“reasonable”, depending on “evolving circumstances.”
LeBel, Fish, and Abella (dissent): Binnie’s approach is too uncertain. Detainee should
be given chance to speak to counsel whenever s/he asks.
Conclusion: All of Sinclair’s statements were ruled to be admissible because they were
given voluntarily. There was no material change to the circumstances, and as a result, the
police had no obligation to provide further council.

37
13.  Bail  and  Other  Pre-­‐Trial  Matters  
Intro  to  Bail,  Multiple  Infos,  Waivers  and  Parties  
• Bail: (Judicial Interim Release). Can occur w/or w/o conditions and sureties. If
the court doesn’t release on bail it means the accused is detained pending trial, and
generally a detention order can only be reviewed by a justice of the Superior
Court.
• This decision IN or OUT is critical, the whole case turns on this:
o IN: then in until trial, @ Remand center, trial date set about 3 months
ahead.
o OUT: easier for the lawyer, can consult more often.
• Informations: occur when a police give a report to Crown Counsel, then they
apply and if they want to prosecute then they swear an information. A person can
be subject @ one time to 1+ informations, in one or more jurisdiction. Each
prosecution remains separate, bail specific to each, and they can be detained on
one released on others.
• Waiver: if the accused pleads guilty to one or more counts the information may
be waived to another jurisdiction for plea and sentencing. Possible to change your
mind about pleading guilty but then the informations go back to their original
jurisdiction.
• Law of Parties: a person can be charged under the law of parties covered in s.21
which states that everyone is a party to an offence who:
o Actually commits it,
o Does or omits anything to do for the purpose of aiding any person to
commit it, or
o Abets any person in committing it.
o Or common intention (counseling) where 2 or more parties form an
intention in common to carry out an unlawful purpose and in doing the
common purpose commit an offence, then both are parties.
R v Dunlop and Sylvester SCC 1979 (Who is a party to a crime)
Ratio: Dickson “Mere presence at the scene of a crime is not sufficient…Something
more is needed: encouragement…” Evidence of involvement, aiding or abetting is
necessary to raise the status to a party.

Facts: Two men convicted of being parties to gang rape, on basis of their being
bystanders but failing to intervene and stop it.

Issue: Are you a party to a crime if you were a mere bystander (failed to stop it or leave)?

Conclusion: Evidence was lacking, both were acquitted.

Bail  (Judicial  Interim  Release)  


• The general idea of s.515 is that the accused shall be released unless the Crown can
show cause why detention (or conditions) are necessary.

38
• There is a preference for release/minimal conditions, the exception is in s.516 (3 day
adjournment)
• Bail hearings are held in Prov. Court except for serious offences (Superior Court). If
they take place after hours they may be w/a JP
• Some possible conditions are: no alcohol, no contact order, no go, curfew, surrender
passport, report to bail supervisor regularly.
• Test for Bail from 515(10): (bail wouldn’t be granted if one of these)
1. PRIMARY GROUND: to ensure attendance in order to ensure they show up
in court (ie) they haven’t shown up before)
2. SECONDARY GROUND: ensure the public’s safety, including the victim
w/regard to circumstances surrounding the arrest and avoid them recommitting
an offence b/c of substantial likelihood they reoffend.
3. TERTIARY GROUND: necessary for the administration of justice, strength
of the Crown’s case, gravity of the offence, circumstances and the fact that the
accused is liable for a long time (rarest ground)

Onus,  Burden  of  Proof,  Standard  of  Proof  and  Breaches  of  Bail  
• Onus to show why you should be detained is on the Crown on a balance of prob.
They can admit any evidence under s.518(1)(e), that the judge considers credible.
• There is a reverse onus if you are in for an indictable offence (ie murder)
• A detention order can be undone:
1. A) @ end of prelim or once trial begins, ask the trial judge to review
(s.523(2)(a)), if you wait 3-4 mths, Crown asks for more time you consent to the
adjournment and ask for bail to be reviewed.
2. B) after 30 days for summary of 90 on indictable, if trial hasn’t begun.
3. C) w/consent of Crown @ anytime, ie) in spousal assault.
4. D) by application to the SCC under s.520, by going upstairs you want the
detention order reviewed immediately.
• If there is a breach of bail look to s.524(1) & (4) for what the Crown will/can do.
• There are other consequences of breach under s.145(3):
• Bail under the Youth Criminal Justice Act: Covered under s.29, can’t detain for
reasons like they don’t have parents. For youths s.29(2) is the rebuttable
presumption that they shouldn’t be detained unless they could be sentenced
under s.39(1)(a).
• S.39 covers when the Crown should commit a young person to custody under
s.42 (youth sentences).

Arraignment  and  Plea  


• @ the arraignment hearing the information and charges are formally read to
accused and the plea is entered.
• Pleas: autrefois acquit (I have already been convicted or acquitted of an
offence/crime for matters arising out of this (double jeopardy rule engaged), not
guilty (not a claim of innocence, a demand that the Crown prove all elements of
offence or disprove valid defences), not criminally responsible due to mental
disorder NCRMD

39
• Here the issue is whether the mental disorder defence applies, if so, then decide
whether they should remain in custody, released on terms or released
unconditionally, judge may not find you fit to stand trial.
• Guilty (can be withdrawn if valid grounds (Adgey). Valid grounds may include;
improper pressure, Crown failing to disclose relevant evidence before the please,
constitutional violation.

Elections,  Preliminary  Hearings,  and  Delay  


• If you plead guilty you go to the sentencing hearing right away.
• There are 3 kinds of offences: summary (up to 6mths in jail and/or $2k fine),
indictable (no limit), hybrid.
• Summary offences are exclusively heard by Prov Court, indictable offences are
either Prov Court (s.553) or Superior Court (s.469), if in neither s. then accused
elects.
Elections: For an indictable offence not in 553 or 469, choice to elect:
1. Prov Court w/out jury or prelim
2. By judge in Superior Court alone, w/prelim (when unsympathetic facts)
3. By a judge & jury in Superior Court, w/prelim (sympathetic cases)
• Possible to re-elect up or down. Also the Crown prefers indictment where the matter
goes directly to the Supreme Court for trial.
Preliminary Hearings: trial in Prov Court to decide whether there is sufficient evidence
which could result in a conviction (low standard), if so then the accused is committed to
trial. Prelim court is not a court of competent jurisdiction, in accordance w. 24 of Charter
evidence can’t be challenged or excluded.
• You might choose a prelim to buy time, get information about Crown’s case.
i) R v Morin SCC 1992
Rule/Ratio: Purposes of s.11(b) are to protect, security of the person, liberty, and
fair trial. Courts must weigh: length of delay, waiver of time period, reasons for
delay, prejudice to accused.

Facts: Accused is charged w/impaired driving, asks for the earliest possible trial date, it is
14.5 months from then, told that is the earliest she can get. Argues that right to be tried in
a reasonable time under s.11(b) is violated, asks for a judicial stay.

Issue: How should courts decide when 11(b) has been violated?

Analysis/Conclusion: s.11(b) protects right to a fair trial balancing societies interests


w/the accused interests. The majority finds she does have a prima facie case for delay but
11(b) wasn’t violated b/c there was no prejudice (substantial). Appeal dismissed.

40
14.  Sentencing  
Mechanics  of  Sentencing:  Range  and  Options  
Step 1: Determine the range of sentence for the offence (how high/low you can go)
(what kind of offence is it)
1. Offence provisions are in the Code, statutes etc.
2. Summary offences are s.787; fine up to $5k up to 6 months or both
3. Indictable offences can include minimum or maximum sentences.
4. Hybrid: Crown can choose at outset whether to proceed as summary or
indictable, can be changed up until the trial date. Crown must elect.
5. Crown will often provide an initial sentencing position (ie if you plead no we
will as for a #>sentence, reduces the # of trials).
6. The Crowns position is based on the state of evidence and the assumption of an
early guilty plea, can depart from here once trial begins.
Step 2: Determining the kind of sentence for an offence: wide range of options
1. A discharge, although found guilty instead of conviction you are discharged, can
be absolute or conditional. Either form chosen must be in the accused’s best
interest and not contrary to the public interest.
2. Suspended sentence, probation order, can be imposed alone or in addition to jail
up to 2 yrs. A breach of probation is a separate offence.
3. Fine alone, still comes w/criminal record.
4. Conditional sentence, “house arrest,” where someone would otherwise be
sentenced to imprisonment where certain conditions are met they can serve the
sentence in the community. Not available where there is a mandatory minimum.
5. Period of imprisonment, actual time in custody depends on federal or prov
legislation. Before 2009 credit for everyday before trial was 2 days. Most
offenders eligible to apply for parole at 1/3 and mandatorily released on parole
after 2/3. The accelerated parole review is available 6 moths before parole
eligibility date or 6 months into sentence whichever is longer.
At Sentencing: s.724 sets out the fact-finding requirements for sentencing. Crown
must prove aggravating facts BARD, other’s on a balance of probabilities.

Principles  of  Sentencing  


• Look to s.718 for the principles; denounce, deter, separate, rehabilitate, reparations,
responsibility
• S.718(1) is principle that sentences must be proportionate to the gravity of the offence
and degree of responsibility of the offender.
• S.718(2) outlines other principles including s.718(2)(e) (Aboriginal)
i) Gladue v The Queen SCC 1999
Rule: When applying s.718(2)(e) the sentencing judge must consider, the unique
background or systemic factors, the types of sentencing procedures and sanctions
that may be appropriate in context. Also always applied to Aboriginal offenders.

41
Facts: Gladue an Aboriginal woman living in an urban center is intoxicated and accuses
her common law husband of cheating. There is an altercation and she stabs him and kills
him. She is sentenced to 3 years.

Issue: How should s.718(2)(e) be applied?

Analysis/Conclusion: Section is remedial, it is meant to remedy the problem of over


population of Aboriginals in the jail system. Court finds that she may have been affected
by systemic factors (but they weren’t brought up). While they find the trial judge erred in
not applying this they find that the offence was serious so the sentence stands. Really the
case came forward to look at the application of s.718, sets the test.
ii) R v Ipeelee and R v Ladue SCC 2012
Rule: The offender doesn’t have the burden of proving systemic factors under
s.718(2)(e) and principles from Gladue do not only apply to serious/violent offences.

Facts: Two Aboriginal offenders out on long-term supervision orders make minor
infringements. They are sentenced to 3 years.

Issues: 1) Do aboriginal offenders bear an evidentiary burden to establish a causal link


between background factors and the offence? Does Gladue apply to serious offences?

Analysis/Conclusion: The courts gave little consideration to background factors therefore


the sentences were reduced.
iii) The Queen v L.M SCC 2008 (Test for when a sentence can be varied)
Rule: Appellate courts can vary a sentence when: it is demonstrably unfit, or if trial
judge erred in principle (failed to consider a relevant factor or over emphasized)

Facts: Accused convicted of sexual assault of young daughter. Not his first offence. He
was given the maximum sentence totaling 15 years and supervision for 10. Court of
Appeal reviewed and reduced his sentence.

Issue: When can an appellate court vary a sentence imposed @ trial level? When should a
maximum sentence be imposed?

Analysis/Conclusion: Court of Appeal erred here by revisiting the sentence w/o it being
unreasonable or in breach of a sentencing principle. Also the sentence imposed was
proportionate in it took notice of mitigating and aggravating circumstances. The trial
judge’s sentence is restored, there were no grounds to vary it.
iv) R v Smith SCC 1987 (When is there a violation of s.12)
Rule: S.12 violated where the sentence is so unfit with regards to the offence and the
offender as to be grossly disproportionate. Gross disproportionality assessed
considering: gravity of offence, personal characteristics, facts, necessary to
rehabilitate, deter, and protect.

42
Facts: Accused is caught with a minimal amount of marijuana coming back into Canada
he has no history and is given the minimum 7 year sentence. Challenges violation of s.12.

Issue: When will a mandatory minimum violate s.12?

Analysis/Conclusion: The minimum in this case is found to fail the minimum


impairment test, the sentence is grossly disproportionate so prima facie it violates s.12 it
also can’t be justified under s.1 b/c doesn’t pass the minimum impairment requirement.

Juries  
The Origin of Juries
• Developed in England in the middle ages. Originally charged with going out and
finding the facts, not the passive role they have today (self-informing jury)
• At the Assize of Clarendon the central government asserted the power of the state.
And all the older methods of trial were abolished. Everything would be done through
court officials.
The Modern Jury
• An accused is entitled to a jury trial for all indictable offences except those within
provincial jurisdiction. Typically only used in murder trials in Canada.
• Tuprin held that a mandatory jury trial doesn’t violate s.11(f) of the Charter.
Selection
• The modern jury consists of 12 members; cannot fall below 10 (if members have to
leave), and is chosen from an array from that area.
• A person will be disqualified from selection if they are an elected representative, a
member of the legal community, have been convicted of an offence in the last 5 years,
are over 65 yrs. Therefore it can be difficult finding a sufficient # of people.
• The qualifications to serve as a juror are determined by the province. The biggest
qualifications are that you are a citizen, a resident and of the age of majority.
• The array is governed by the provincial government by the prov. Jury Act. The federal
government however is responsible under the Code for the selection of a Petit jury.
• The array is made up of 60-180 qualified people who are summoned, using voter lists
and band lists (in BC).
• There have been few challenges to the array process; in R v Butler and R v Kent the
array was challenged b/c of exclusion of Aboriginal people.
• From the array the petit jury is selected under the Criminal Code, they are essentially
names chosen out of a box.

Challenges  
Peremptory
• Under s.634 the Crown or Accused can simply “challenge” jurors, they don’t have to
give any explanation or reason.
• The number of challenges allowed depends on the offence.
• In Canada jurors are presumed to be impartial R v Sherratt.
Challenge for Cause
• These challenges have a narrower scope but are unlimited.

43
• The presumption of impartiality can be rebutted using s.638(1)(b) which allows you to
charge someone for cause b/c the juror isn’t indifferent.
• Under this section presumption is displaced that a juror is “indifferent”
Adverse Policy
• In exceptional circumstances a judge may permit counsel to as a number of
preapproved questions. They will have a threshold to only allow legitimate concerns.
• Regina v Find: the court held that questions based solely on the nature of the offence
weren’t permitted. The evidence of impartiality of the jurors b/c of the bias as a result
of the reaction to the offence was not sufficient.
• R v Parks: the defence tried to challenge jurors based on racial bias against black
people in downtown Toronto. The court held that this challenge was allowed.
• R v Williams: followed after Parks the court held that the trial court had erred in not
allowing a challenge for cause based on discrimination. They applied a two part test;
o Behavior bias: is there a reason to suppose the jury pool might contain people
who are prejudiced and who might not be able to set their prejudice aside
should be excluded.
o Attitudinal bias: How will their prejudice play out in the context of the trial,
will their bias affect the process, this will be determined in the questioning
process. (Generally the 1st can be assumed the 2nd not always).

The  Role  of  Counsel  


i) R v Boucher SCC [1955] (Crown told jury he was guilty, Crown is not out to obtain a
guilty verdict)
Rule: The comments to the jury were held improper. The purpose of a criminal
prosecution is not to obtain a conviction; it is to lay before a jury what the Crown
considers to be credible evidence relevant to the alleged crime. This should be done
firmly and to its ultimate strength but also fairly (Rand J).

Facts: Boucher is found guilty of murder, he appeal this based on the direction the jury is
given at trial. Right before the end of the trial the Crown told the jury if Boucher wasn’t
guilty they wouldn’t have gone this far, “you can trust us he’s guilty.”

Issue: Whether it is appropriate for the Crown to express the view that Boucher was
guilty? If the role of the Crown to obtain a conviction? If not what is it?

Conclusion: The trial was rendered unfair and the conviction set aside.

Rules of the Professional Conduct on the Role of the Crown:


From the Law society of BC’s Professional Conduct Handbook: “When engaged as a
prosecutor the lawyer's prime duty is not to seek a conviction, but to see that justice is
done.”
ii) R v Stinchcomb SCC [1991] (the Duty to disclose all relevant information)
Rule: The Crown has a general duty to disclose all relevant information (in its
possession) whether or not it intends to use it. Relevant refers to anything potentially

44
exculpatory or inculpatory. This obligation is also ongoing. Though there are two
exceptions: privileged information and anything clearly irrelevant.

Facts: A lawyer was charged with fraud, breach of trust and theft. The police obtained a
statement from his former assistance, the Crown then decided not to use it or call on the
assistant. They disclose the existence of the statement but not the content. The defence
made requests for the statement but is refused, the judge affirms this. Stinchcomb is
convicted.

Issue: Whether the Crown is obliged to disclose the statement to the defence? Does the
Crown have to disclose evidence it doesn’t intend to use?

Analysis/Conclusion: The court can review the Crown’s discretion about what
information it excepts. Also the defence doesn’t have a reciprocal obligation except in
reference to alibi’s which it must disclose if it intends to use them.

Role  of  Defence  


• Lawyer has a duty to protect the client as far as possible from being convicted
except by a competent court/jurisdiction, notwithstanding the lawyers personal
opinion.
• A lawyer can only defend two people in a trial if there is no conflict of interest.
• Admissions to your lawyer impose strict limitations on the lawyers ability an it is
the lawyers responsibility to make the client aware of this.
• Ie) if the accused admits guilt the lawyer cannot deny this evidence in court by
suggesting otherwise.

45

You might also like